Download as docx, pdf, or txt
Download as docx, pdf, or txt
You are on page 1of 71

Page | 1

Permutation and Combinations: Art of Counting

Combinatorics
The Art of Counting
Best way to learn permutation and
Combination.

MathsExplorer.com Mathsclub.org where Mathematics is more colourful

Page | 2

Permutation and Combinations: Art of Counting

Contents

Contents

Page Numbers

Definition

2-4

Fundamental Principle of Counting

4-6

Multiplication Principal

6-10

Addition Principle

10-13

Bijection Principle

13-14

Inclusion-Exclusion Principle

14-16

Factorials

15-16

Permutations

16-19

Circular Permutations

19-33

Permutations involving Identical Objects

33-40

Exponent of Prime p in n!

40-43

Assignments (Subjective)

43-45

Assignment level II

45-49

Assignments (Objective)

49-74

MathsExplorer.com Mathsclub.org where Mathematics is more colourful

Page | 3

Permutation and Combinations: Art of Counting

Combinatorics :The Art of Counting

Introduction:.Combinatorics deals mainly with questions focused on counting. Questions like In how many
ways is it possible to...? or What is the possible number of ?
Here are a few examples
1. Bar Codes

You must have observed these stamped on almost everything you buy!
How many such different barcodes are possible?
How could the barcode be designed or modified so that there can be at least 1000000 different codes?

2. Vehicle Registration Numbers

How many vehicles can be registered, given the format of the registration number on the left?
10000? 1000000? 99999999? What do you feel it to be?

3. p@sSwoRd$

How much time would it require (worst case) to guess (or hack) a password or a secret code, which is 9
character long and can contain alphabets, numbers and some special characters like @#$%&*?
Well, one thing it will surely depend upon is How many such different codes would be possible?
4. Smartphone Lock Screens

MathsExplorer.com Mathsclub.org where Mathematics is more colourful

Page | 4

Permutation and Combinations: Art of Counting

This looks familiar, doesnt it?


Suppose you forget the code to unlock your screen, and you start guessing the code trying all the possible
combinations (assuming there is no other way!).
How much time will it take you to guess the correct code?
Once again How many different unlock codes could be possible?
Hmm.. Thats a tougher one!
Problems such as these require a systematic and a logical way of counting to be answered correctly, and, in a
reasonable amount of time. This topic is all about that to count without counting!
Well begin with the basic principles of counting, and as we proceed, well develop some advanced techniques
which can help us answer a lot of complex counting related problems and ultimately count the uncountable!
Permutations & Combinations
Fundamental Principle of Counting
There are two basic principles of counting The Multiplication Principle and The Addition Principle.
All subsequent concepts, (and formulas) in Permutations & Combinations will build upon these two principles,
which are pretty simple to grasp.
To introduce the principles, lets consider a simple problem:
Suppose a car company sells the following categories of cars

Hatchback

Sedan

Each of which comes in 3 different models Standard, Deluxe and Sports

MathsExplorer.com Mathsclub.org where Mathematics is more colourful

Page | 5

Permutation and Combinations: Art of Counting

A simple question: How many different types of cars does the company sell?
Before reading further, think about it for a minute and try to find out the answer yourself (and of course, dont
worry about getting it wrong)
Let us list all the possible cases.
By counting all the cases, we can see that there are 6 different types of cars in all. (Did you get it right?)
Here is how we can reach that number by a simple calculation.
Since there are 2 car categories (hatchback & sedan), and each of these comes in 3 different models, the total
number of types of cars can be obtained multiplying the number of categories (2) with the number of models
available for each category (3), which is 2 x 3 = 6.
(In other words there will be 3 hatchback models + 3 sedan models = 2 x 3 = 6 total types of cars)
Lets complicate the problem a bit. Suppose each of the above models were available in 2 different colours.

How many different cars do you think are available now? Lets count again.
This time, there are 12 different types of cars in total.
Since each of the car is available in two colours, and there are 6 different types of cars (as calculated above),
we have 6 blue cars + 6 black cars = 6 x 2 = 12 cars.
And this number can be obtained by multiplying together, the number of categories of cars (2), number of
models for each category (3) and the number of colours available for each model (2): 2 x 3 x 2 = 12.
And what if there was another colour (say red) available for each model? The total number of cars would now be
obtained by multiplying 2 (categories), 3 (models per category) and 3 (colour per model), which equals 18 (2 x
3 x 3).
(Note that this is 6 more than the number 12 obtained previously, as there would be 6 more red cars)
Make sense? Try listing these 18 cases to make sure everything is all right.
Ill stop for now. In the next part, well move on to the Multiplication Principle, and see some of its applications.
(PS: Weve already used the Multiplication Principle thrice in this part!)
Multiplication Principal:
Now that youve got some idea of counting related problems and use of multiplication to answers them, Ill now
state the Multiplication Principle a bit formally:
Definition: If a collection of objects can be separated into m different types, and each of these types can be
separated into n different subtypes, then there will be m x n different types of objects in all.
(There is a reason why Ive been bolding each each along the way, because each each matters and things will
be different without them. Ill come to this a little later.)

MathsExplorer.com Mathsclub.org where Mathematics is more colourful

Page | 6

Permutation and Combinations: Art of Counting


To be more precise and define above multiplication principle mathematically,
Definition: If one experiment has n possible outcomes and another experiment has m possible outcomes, then
there are m n possible outcomes when both of these experiments are performed.
In other words, if a job has n parts and the job will be completed only when each part is completed and the first
part can be completed in a1 ways, the second part can be completed in a2 ways and so on . . . the nth part can
be completed in an ways, then the total number of ways of doing the job is a 1a2a3...an. This is known as the
Multiplication principle.
For our cars example, we could separate the cars into 2 types (sedan and hatchback) and each of these could
be separated into 3 subtypes (models), making 2 x 3 = 6 total types of cars.
(We could also have gone the other way round and first separate the cars as models, and then further separate
these into sedans and hatchbacks. The answer would have remained the same: 3 x 2 = 6)
Further, each of these 6 cars can be separated into 2 types (red and blue) making a total of 6 x 2 = 12 different
types of cars.
The multiplication rule is not just limited to classification of objects. It can also be applied in different contexts.
Consider this problem, for example If there are 3 different flights from A to B and 2 different trains from B to C,
in how many different ways can a person reach from A to C (using only these flights and trains)?
Here are all the cases for you to look at.
The possible routes are 1a, 1b, 2a, 2b, 3a, and 3b a total of 6.
Again, for each route from A to B, there are 2 routes from B to C. And since there are 3 routes from A to B,
there will be a total of 3 x 2 different routes in all.
In other words, the number of ways will be obtained by multiplying the number of routes from A to B (3) with the
number of routes from B to C (2) available for each route from A to B: 3 x 2 = 6.
Heres another way we can state the multiplication principle:
If a task T can be divided into subtasks T1 and T2, which can completed in m ways and n ways respectively, and
T will be completed by completing both T1 and T2, then the number of ways of completing T will be m x n
Lets think of this example again.
The two subtasks T1 and T2 can be thought of as reaching from A to B and reaching from B to C respectively,
where T1 can be completed in 3 ways and T2 can be completed in 2 ways.
Since the task of reaching from A to C will be completed by completing both T1 and T2, therefore the number of
ways of reaching from A to C will be 3 x 2 = 6.
Heres another simple problem.

In how many ways can you reach from A to D, given the possible routes from A to B, B to C and C to D?
The task here is A to D, which will be completed by completing the tasks A to B and B to D.
Here B to D can further be divided into subtasks B to C (2 ways) and C to D (3 ways), both of which must be
completed. Therefore the number of ways of reaching from B to D is 2 x 3 = 6.
And the number of ways of completing A to D will be 3 (A to B) x 6 (B to D) = 18 ways.
We could also directly write this number as 3 (A to B) x 2 (B to C) x 3 (C to D) = 18 ways.

MathsExplorer.com Mathsclub.org where Mathematics is more colourful

Page | 7

Permutation and Combinations: Art of Counting


To extend the Multiplication Principle:
If a task T can be divided into n subtasks T1,T2,T3,.Tn, which can completed in m1, m2, m3 mn ways
respectively and T will be completed by completing each of these subtasks, then the number of ways of
completing T will be m1 x m2 x m3 x . x mn
To elaborate above principle here are few solved examples, try to solve them yourself before seeing the
solution.
Example1:

A college offers 7 courses in the morning and 5 in the evening. Find the possible number
of choices with the student if he wants to study one course in the morning and one in the
evening.

Solution:

The student has seven choices from the morning courses out of which he can select one course
in 7 ways.
For the evening course, he has 5 choices out of which he can select one in 5 ways.
Hence the total number of ways in which he can make the choice of one course in the morning
and one in the evening = 7 5 = 35.

Example 2:

A person wants to go from station A to station C via station B. There are three routes from
A to B and four routes from B to C. In how many ways can he travel from A to C?

Solution:

A B in 3 ways
B C in 4 ways
A C in 3 4 = 12 ways
Remark:

The rule of product is applicable only when the number of ways of doing each part is independent of
each other i.e. corresponding to any method of doing the first part, the other part can be done by any
method.

Example 3:

Find the number of three-digit natural numbers having digits in increasing order from left
to right.

Solution:

Here we have to fill three places.


First place can be filled by numbers 1, 2, .7,
second by 2, 3, .8 and third by 3, 4, .9 . i.e.
the number of ways of filling each place is
seven but total number of ways is not 7 77 =
343.
Reason behind this is that corresponding to 1 at first place, second place can be filled up by
anyone of the seven digits 28 but when we put 2 at first place, the number of ways of filling
second place is only six. So the number of ways of doing each part is not independent. So rule
of product is not applicable in this case. The right approach for this problem is that first select

MathsExplorer.com Mathsclub.org where Mathematics is more colourful

Page | 8

Permutation and Combinations: Art of Counting


three distinct non-zero digits which can be done in 9C3 ways, then arrange them in increasing
order which can be done in one way only. Therefore, the required number of natural numbers is
9
C3 1 = 84.
Example 4:

How many (i) 5-digit (ii) 3-digit numbers can be formed by using 1, 2, 3, 4, 5 without
repetition of digits.

Solution:

(i) Making a 5-digit number is equivalent to filling 5 places.


Places:

Number of
Choices:

The first place can be filled in 5 ways using anyone of the given digits.
The second place can be filled in 4 ways using any of the remaining 4 digits.
Similarly, we can fill the 3rd, 4th and 5th place.
No. of ways of filling all the five places
= 5 4 3 2 1 = 120
120 5-digit numbers can be formed.
(ii) Making a 3-digit number is equivalent to filling 3 places.
Places:

Number of
Choices:

Number of ways of filling all the three places = 5 4 3 = 60


Hence the total possible 3-digit numbers = 60.

Example 5:

How many 4-letter words can be formed using a, b, c, d and e


(i) Without repetition

Solution:

(ii) with repetition.

(i) The number of words that can be formed is equal to the number of ways of filling the four
places.
Places:

Number of
Choices:

5 4 3 2 = 120 words can be formed when repetition is not allowed.

MathsExplorer.com Mathsclub.org where Mathematics is more colourful

Page | 9

Permutation and Combinations: Art of Counting


(ii) The number of words that can be formed is equal to the number of ways of filling the four
places.
Places:

Number of
Choices:

First place can be filled in 5 ways. If repetition is allowed, all the remaining places can be filled in
5 ways each.
5 5 5 5 = 625 words can be formed when repetition is allowed.

Fundamental Principle of Counting: Addition Principle


Hope that you now have some idea of the multiplication principle. This lesson will be focused on another basic
principle of counting, known as the Addition Principle.
Lets start with a simple problem: Suppose there are 3 different flights and two different trains connecting two
places A and B. In how many ways can you reach from A to B (using only these flights or trains)?

The answer seems obvious: 5


Why? Because you can either choose one of the flights (3 choices) or choose one of the trains (2 choices).
Therefore the total number of choices are 3 + 2 = 5 (Flight 1 or Flight 2 or Flight 3 or Train 1 or Train 2)
Another one: A restaurant offers 4 non-veg dishes and 3 veg dishes as starters. How many different types of
starters does the restaurant offer?
Again, the answer will be 4 + 3 = 7.
So what is the addition principle about? Here is one way to state it:
If two events A1 and A2 can occur in m and n ways respectively (none of these being common), then either of
these events can occur in m + n ways.
An analogous statement for the multiplication principle will be:
If two events A1 and A2 can occur in m and n ways respectively, then both of these events can occur together in
m x n ways.
Now that weve some idea about the multiplication and the addition principles of counting, lets dig a little deeper
to understand them better.
Let us come back to the cars example again.

MathsExplorer.com Mathsclub.org where Mathematics is more colourful

P a g e | 10

Permutation and Combinations: Art of Counting


Suppose the Sports model exists only for the sedan and not for the hatchback. How many different types of cars
are there now?
We can see that there are now 5 different types of cars, instead of 6 2 different hatchbacks and 3 different
sedans.
In this case we cannot apply the multiplication principle directly, because each of the categories is not available
in all the three models. Therefore we have to separate out the cases and count them individually.
Lets bring in the colours now. Suppose all models are available in both blue and black. How many different cars
would be there?
As you can see, there are 10 different cars.
In this case, well have to use both the addition and the multiplication principles together.
There would be 2 x 2 = 4 hatchbacks, and 3 x 2 = 6 sedans, making a total of 10 different types of cars.
Or, total number of cars = (2 x 2) + (3 x 2) = 10.
Lets complicate the problem a little more.
Suppose that the Deluxe and the Sports versions of the sedan category are also available in white. How many
different cars are available now?
This time the total can be calculated as (2 x 2) + (1 x 2) + (2 x 3) = 12.
2 x 2 = 4 hatchbacks. And (1 x 2) + (2 x 3) = 8 sedans. A total of 12.
Thus, whenever the things to be counted need to be separated into cases, we count the cases separately and
add them together.
To be more precise and define above multiplication principle mathematically,
Definition: If one experiment has n possible outcomes and another has m possible outcomes, then there are (m
+ n) possible outcomes when exactly one of these experiments is performed.
In other words, if a job can be done by n different methods and for the first method there are a 1 ways, for the
second method there are a2 ways and so on . . . for the nth method, an ways, then the number of ways to get the
job done is (a1 + a2 + ... + an).
Fundamental Principle of Counting: Few Examples
This lesson will cover a few examples to help you understand better the fundamental principles of counting.
Question: Find the number of 3-digit numbers formed using the digits 3, 4, 8 and, 9, such that no digit is
repeated.
Solution: The task of forming a 3-digit number can be divided into three subtasks filling the hundreds place,
filling the tens place and filling the units place each of which must be performed to complete the task.
Now there are 4 ways to complete the first subtask (i.e. filling the thousands place), since we have 4 digits to
begin with. Next, there are 3 ways to complete to fill the tens place (the second subtask), as now there are only
3 digits left to be used (repetition of digits isnt allowed). Lastly, there are 2 ways to complete the third task.
Therefore the total number of ways to complete the task (which equals the required number of 3-digit numbers)
will be 4 x 3 x 2 = 24.
Ill list out the 24 numbers, in case you dont believe me.
348 349 384 389 394 398 438 439 483 489 493 498 834 839 843 849 893 894 934 938 943 948 983 984

MathsExplorer.com Mathsclub.org where Mathematics is more colourful

P a g e | 11

Permutation and Combinations: Art of Counting


Note that the digits could have been filled in any order, not necessarily in the order which I did. The answer
obtained would have been the same.
Q2. How many 3-digit numbers can be formed using the digits 4, 5 and 6, such that the digit can repeat?
Soln. This seems similar. But this time the number of ways to fill the hundreds, tens and the units place would
be 3 in each case, (and not 3, 2, 1) because the digits are allowed to be repeated.
Therefore the number of such 3-digit numbers would be 3 x 3 x 3 = 27. Ill list them out again.
444 445 446 454 455 456 464 465 466 544 545 546 554 555 556 564 565 566 644 645 646 654 655 656 664
665 666
Okay. Lets move on to a little more complex problems. Ill stick to counting numbers in this lesson.
Q3. Find the number of 5-digit even numbers formed using the digits 1 to 9, such that repetition of digits is
(i) allowed (ii) not allowed
Soln. I suggest that you grab a pen / paper and try to compute the answer yourself before reading further.
(i) In this case we have a restriction on filling the units place, as the number needs to be even. There are only 4
ways to do so (using 2, 4, 6 or 8). For the other four places, we have 9 choices each (as repetition is allowed).
Therefore the number of ways will be 9 x 9 x 9 x 9 x 4 = 32805
(I wont list the numbers this time. You have no option but to believe me.)
(ii) This one is a little tricky. You might say that the answer is 9 x 8 x 7 x 6 x 4, as there will be 4 ways to fill the
last digit (using 2, 4, 6 or 8).
But theres a problem. Were not sure that after filling the first four places (from the left), whether well be left with
4 choices for the units place or not. Because one of the even digits might have already been used before we
reach the units place.
To get around this problem, we fill the units place first using the even digits, followed by 8, 7, 6 and 5 choices
respectively for the remaining places.
Therefore the number of such 5-digits numbers will be 4 x 8 x 7 x 6 x 5 = 6720.
Q4. Find the number of 4-digit odd numbers formed using the digits 0 to 9 such that repetition of digits is
(i) allowed (ii) not allowed
Soln. The units digit in this case can be filled in 5 ways (using the odd digits). Now, there are only 9 ways to fill
the thousands place, as 0 cannot be used there. The remaining places can be filled 10 ways each.
Therefore the total numbers in this case would be 9 x 10 x 10 x 5 = 4500.
(Note that this is half of the total 9000 4-digit numbers possible, the other half being even)
(ii) Well first fill the units place with an odd digit 5 ways. Next we have the thousands digit 8 ways (leaving
out the 0). Finally, 8 ways for the hundreds place (the 0 is back) and 7 ways for the tens place. The total number
will be 5 x 8 x 8 x 7 = 2240
I guess thatll be it for this lesson. As a practice problem try and count the number of 5-digit even numbers using
the digits 0 to 9, with and without repetition.
Illustration6: Acollegeoffers7coursesinthemorningand5intheevening.Findthenumberofwaysastudentcan
select exactly one course, either in the morning or in

theevening.
Solution:

Thestudenthassevenchoicesfromthemorningcoursesoutofwhichhecanselectonecoursein7ways.
Fortheeveningcourse,hehas5choicesoutofwhichhecanselectonecoursein5ways.

MathsExplorer.com Mathsclub.org where Mathematics is more colourful

P a g e | 12

Permutation and Combinations: Art of Counting


Hencehehastotalnumberof7+5=12choices.
Example:

A person wants to leave station B. There are three routes from station B to A and four
routes from B to C. In how many ways can he leave the station B.

Solution:

B A in 3 ways
B C in 4 ways
He can leave station B in 3 + 4 = 7 ways.

Another beautiful principle


Bijection Principle:
Generally it is not easy to count the numbers belonging to a very large collection (set) directly. Let us assume
that we have a set X and we want to count n(x) ( number of elements in X). And we have another set Y
and n(Y) can be counted (easily). Now if there exists one-one and onto relationship from set X to set Y, then n(X)
= n(Y).
In other words, if X and Y are two finite sets such that there is a bijection ( i.e. one-one onto map) from X to Y ,
then n(X) = n(Y).
For example let us take a simple case. The sitting capacity of a class is 30. On a particular day, the class is full
then, without actual counting, we can say that the total number of the students present in the class is 30.

MathsExplorer.com Mathsclub.org where Mathematics is more colourful

P a g e | 13

Permutation and Combinations: Art of Counting


Inclusion-Exclusion Principle:
Let U be a finite set of m elements and p 1 , p2 , p3 , . . . ,p r be some properties which the objects of U may or
may not have, then the number of elements of U which have at least one of the properties p 1, p2, p3 , . . . ,pr is
given by
r

n
A i
i1
= S1 - S2 + S3 S4 + . . . +(1)r-1Sr .
and the number of elements of U which have none of the properties p1, p2, p3 , . . . ,pr is given by
r c
n
A i
i1

= m S1 + S2 - S3 + . . . + (-1) r Sr .
A ic
Where Ai denotes the subset of U each of whose elements has the property p i( i = 1, 2, . . . , r) and
denotes
the subset of U none of those elements has the property pi
Ai A j denotes the set of all elements of U having both properties pi and pj and so on. And Sk denotes the total
number of elements of U having any k of the given r properties.
E.g. S1 = n(A1) + n(A2) +...+ n(Ar),
S2 = n(A1A2) + n(A1 A3) +...+ n(A1Ar) + n(A2A3) +...+ n(A2Ar) +...+ n(Ar-1 Ar).
In particular case:

For r =2, n(A1A2) = n(A1) +n(A2) - n(A1A2)


For r = 3,
n(A1A2 A3) =n(A1) +n(A2) +n(A3) n(A1A2) n(A1A3) n(A2A3) + n(A1 A2 A3).
For r = 4,
n(A1A2A3A4) = n(A1) +n(A2) +n(A3) + n(A4) n(A1A2) n(A1A3) n(A1A4) -n(A2A3) n(A2A4)
n(A3A4)+ n(A1 A2 A3) + n(A1 A2 A4) + n(A1 A3 A4) + n(A2 A3 A4) n(A1A2 A3 A4).

a
b

Example:
Find the number of integers between 1 and 1000, both inclusive
Which are divisible by either of 10, 15 and 25.
Which are divisible by neither 10 nor 15 nor 25.
Solution:

Let U denote the set of numbers between 1 and 1000 so that n(U) = 1000. Let A, B and C denote
the sets of integers divisible by 10, 15 and 25 respectively.
1000
10

Then n(A) =
=100,
1000

15

n(B) =
= 66,
1000
25

and n(C) =
= 40 .
S1 = n(A) +n(B) +n(C) = 100 + 66 + 40 = 206
Considering two of the three sets A, B, C, we have
A B = set of integers divisible by 10 and 15
= set of integers divisible by 30 (L.C.M of 10 and 15)
1000
30 33

n( A B) =
.

MathsExplorer.com Mathsclub.org where Mathematics is more colourful

P a g e | 14

Permutation and Combinations: Art of Counting


Similarly, B C = set of integers divisible by 15 and 25
= set of integers divisible by 75 (L.C.M of 15 and 25)
1000
75 13

n( B C) =
.
Similarly, A C = set of integers divisible by 10 and 25
= set of integers divisible by 50 (L.C.M of 10 and 25)
1000
50 20

n( A C) =
.
S2 = 33 + 13 + 20 = 66.
Finally, A B C = set of integers divisible by 10, 15 and 25
= set of integers divisible by 150 (L.C.M of 10, 15 and 25)
1000
150 6

n( A B C) =
.
S3 = 6 .
a) We have to find the number of integers divisible by either
of 10, 15, and 25 i.e. n(A BC)
n(A BC) = S1 S2 +S3 = 206 66 + 6 = 146 .
b) The number of integers divisible by neither 10 nor 15, nor 25
= n(U) n(A BC) = 1000 146 = 854.
Factorials:
The factorial of a natural number n is the product of all natural numbers from 1 to n, that is, the product 1 x 2 x 3
x x n. This product is denoted as n! (i.e. the number followed by an exclamation mark).
For example 4! = 1 x 2 x 3 x 4 = 24, and 7! = 1 x 2 x 3 x 4 x 5 x 6 x 7 = 5040.
As a convention, zero factorial (0!) is defined to be 1. (Ill come back to this later). And you neednt worry about
factorial of negative numbers or fractions.
Note that n! = n x (n 1)! For example 5! can be written as 5 x (4 x 3 x 2 x 1) which equals 5 x 4!
Okay, moving on.

MathsExplorer.com Mathsclub.org where Mathematics is more colourful

P a g e | 15

Permutation and Combinations: Art of Counting


Permutations
The term permutation is used to indicate ordered arrangements of objects. For example, the permutations of the
three letters Q, W and E (in a row) are QWE, QEW, WEQ, WQE, EWQ and EQW.
The number of such permutations will be 3 x 2 x 1 = 3! = 6 (Weve already seen the method of calculation in a
previous lesson)
Similarly if we had four objects to be arranged in a row, for example, forming 4-digit numbers (without repetition)
using 4, 6, 7, and 9, the number of permutations will be 4 x 3 x 2 x 1 or 4!
This number permutations of objects, taken all at a time (without repetition) is denoted as nPn or P(n, n). As you
can see, this number comes out to be n! (n-factorial)
In case we want the number of 4-digit numbers (without repetition of digits) using the digits 1 to 9, this will be 9 x
8 x 7 x 6.
This number is same as the number of permutations of 9 objects, taken 4 at a time. This is denoted as 9P4 or
P(9, 4), which equals 9 x 8 x 7 x 6. The number can be expressed using factorials. If we multiply and divide it by
5 x 4 x 3 x 2 x 1, we get P(9, 4) = (9 x 8 x 7 x 6 x 5 x 4 x 3 x 2 x 1)/(5 x 4 x 3 x 2 x 1) = 9!/5! or 9!/(9 4)!. Neat.
To generalize, the number of permutations of n different objects in a row, taken r at a time is denoted as nPr or
P(n, r) which equals n!/(n r)!
Well, thats it. Things will get better when you go through examples, which Ill cover next. See you soon.
Permutations: Examples
This lesson covers a few examples relating to permutations, particularly those involving nPr. There isnt be
anything new to be covered here. The problems can be solved using the methods weve already seen. This is
just to get you familiarized with factorials and notations.
Q1. Find the number of 4-letter sequences that can be formed using the English alphabets, such that alphabets
are not repeated in any sequence.
Sol. This equals the number of permutations of 26 different objects, taken 4 at a time 26P4. We could also
have computed the answer using the multiplication principle (as we did previously). The answer will come out to
be 26 x 25 x 24 x 23 (which is same as 26P4)
Q2. How many 4-digit numbers can be formed using the digits 3, 4, 5 and 7, using each digit only once?
Sol. The number of numbers in this case would equal 4P4 or 4! number of permutations of 4 different objects,
taken all at a time.
Lets complicate things a bit.
Q3. Find the number of 3-digit numbers formed using the digits 1 to 9, without repetition, such the numbers
either have all digits less than 5 or all digits greater than 4.
Sol. Here, 342 is a possible number, 769 is another, but 385 is not. We can divide the numbers to be counted
into two cases.
For those which have all the digits less than 5, we have to count the number of permutations of 4 digits (1, 2, 3,
and 4), taken three at a time: 4P3.
For those which have all the digits greater than 4, we have to count the number of permutations of 5 digits (5, 6,
7, 8 and 9), taken three at a time: 5P3.
The total number of numbers would be 4P3+5P3, which equals 24 + 20 = 44.
Thats it for now. Next, Ill talk about circular arrangements of objects.
We can do the arrangement by box method also as I am describing as follows,
(a) Arranging n objects, taken r at a time is equivalent to filling r places from n things.
rPlaces:

n-1 n-2 n-3

MathsExplorer.com Mathsclub.org where Mathematics is more colourful

P a g e | 16

Permutation and Combinations: Art of Counting


Number of Choices:

n -(r-1)

The number of ways of arranging = The number of ways of filling r places


= n(n 1) (n 2) .. (n r + 1)

n(n - 1) (n - 2) .. (n - r 1) ((n - r)! )


n r !
=

n!
n r !
=

= nPr

(b) The number of arrangements of n different objects taken all at a time = npn = n!
With Repetition:
(a) The number of permutations (arrangements) of n different objects, taken r at a time, when each object may
occur once, twice, thrice. upto r times in any arrangement
= The number of ways of filling r places where each place can be filled by any
one of n objects .
rPlaces:

Number of
Choices:

The number of permutations =The number of ways of filling r places


= (n)r
(b) The number of arrangements that can be formed using n objects out of which p are identical (and of one
n!
p! q! r!
kind), q are identical (and of another kind), r are identical (and of another kind) and the rest are distinct is
.
Example:

(a) How many anagrams can be made by using the letters of the word HINDUSTAN.
(b) How many of these anagrams begin and end with a vowel.
(c) In how many of these anagrams, all the vowels come together.
(d) In how many of these anagrams, none of the vowels come together.
(e) In how many of these anagrams, do the vowels and the consonants occupy the same
relative positions as in HINDUSTAN.

Solution:

(a) The total number of anagrams

9!
2!
= Arrangements of nine letters taken all at a time =

= 181440.

MathsExplorer.com Mathsclub.org where Mathematics is more colourful

P a g e | 17

Permutation and Combinations: Art of Counting


(b) We have 3 vowels and 6 consonants, in which 2 consonants are alike. The first place can be
7!
2!
filled in 3 ways and the last in 2 ways. The rest of the places can be filled in
ways. Hence the
7!
2!
total number of anagrams = 3 2
= 15120
(c) Assume the vowels (I, U, A) as a single letter. The letters (IUA) H, D, S, T, N, N can be
7!
2!
arranged in
ways. Also IUA can be arranged among themselves in
3! = 6 ways.
7!
2!
Hence the total number of anagrams =
6 = 15120
(d) Let us divide the task into two parts. In the first, we arrange the 6 consonants as shown below
6!
2!
in
ways.
CCCCCC
(C stands for consonants and stands for blank spaces in between
them)
7!
4!
Now 3 vowels can be placed in 7 places (in between the consonants) in 7p3 =
= 210 ways.
6!
2!
Hence the total number of anagrams =
210 = 75600.
(e) In this case, the vowels can be arranged among themselves in 3! =
6 ways.
Also, the consonants can be arranged among
6!
2!
themselves in
ways.
6!
2!
Hence the total number of anagrams =
6 = 2160.
Example: How many 3 digit numbers can be formed using the digits 0, 1,2,3,4,5 so that
(a) digits may not be repeated
d

Solution:

(b) digits may be repeated

(a) Let the 3-digit number be XYZ


Position (X) can be filled by 1,2,3,4,5 but not 0. So it can be filled in 5 ways.
Position (Y) can be filled in 5 ways again. (Since 0 can be placed in this postion).
Position (Z) can be filled in 4 ways.
Hence, by the fundamental principle of counting, total number of ways is

MathsExplorer.com Mathsclub.org where Mathematics is more colourful

P a g e | 18

Permutation and Combinations: Art of Counting


5 x 5 x 4 = 100 ways.
(b) Let the 3 digit number be XYZ
Position (X) can be filled in 5 ways
Position (Y) can be filled in 6ways.
Position (Z) can be filled in 6 ways.
Hence by the fundamental principle of counting, total number of ways is
5 x 6 x 6 = 180.

Example:

Find the number of ways in which 6 letters can be posted in 10 letterboxes.

Solution:

For every letter, we have 10 choices (i.e. 10 letterboxes).


Hence the total number of ways = 106 = 1,000,000

Circular Permutations:
So far, weve only considered arrangements in a line or a row. This lesson will talk about arrangement of
(distinct) objects in a circle.
Consider the problem: In how many ways can 5 persons be seated at a round table? Or, in how many ways can
5 distinct objects be arranged in a circle?
The arrangements differ only in the relative order of the objects, and the relative spacing between the objects
does not matter.
To answer the question, consider a fictitious tire company called ANODE, which has to print its company name
on its tires.
The five linear arrangements ANODE, NODEA, ODEAN, DEANO and EANOD will be identical when arranged
in a circular fashion. Have a look at the figure below.

Each of the tires can be rotated to align with any one of the rest. That means all of these arrangements are
identical, even though we started with different linear arrangements.
Therefore 5 linear arrangements will correspond to only one circular arrangement.
Coming back to the question In how many ways can 5 distinct objects be arranged in a circle?

MathsExplorer.com Mathsclub.org where Mathematics is more colourful

P a g e | 19

Permutation and Combinations: Art of Counting


Now there are 5! or 120 different linear arrangements possible. Therefore the number of circular arrangements
will be 5! 5. (As every 5 linear arrangements will correspond to 1 circular arrangement). This is same as 4! or
24.
To generalize, the number of arrangements of n distinct objects in a circle will be n! / n, or (n 1)!

Circular Permutations: Examples 4

Q1. In how many ways can 6 people be seated at a round table?


Soln. As discussed, the number of ways will be (6 1)!, or 120.
To the next..
Q2. Find the number of ways in which 5 people A,B,C,D,E can be seated at a round table, such that
(i) A and B must always sit together.
(ii) C and D must not sit together.
Soln. (i) If we wish to seat A and B together in all arrangements, we can consider these two as one unit, along
with 3 others. So effectively weve to arrange 4 people in a circle, the number of ways being (4 1)! or 6. Let me
show you the arrangements:

But in each of these arrangements, A and B can themselves interchange places in 2 ways. Heres what Im
talking about:

Therefore, the total number of ways will be 6 x 2 = 12.


(ii) The number of ways in this case would be obtained by removing all those cases (from the total possible) in
which C & D are together. The total number of ways will be (5 1)! or 24. Similar to (i) above, the number of
cases in which C & D are seated together, will be 12. Therefore the required number of ways will be 24 12 =
12.
Another example related to seating
Q3. In how many ways can 3 men and 3 ladies be seated at around table such that no two men are seated
together?
Soln. Since we dont want the men to be seated together, the only way to do this is to make the men and
women sit alternately. Well first seat the 3 women, on alternate seats, which can be done in (3 1)! or 2 ways,
as shown below. (Were ignoring the other 3 seats for now)

MathsExplorer.com Mathsclub.org where Mathematics is more colourful

P a g e | 20

Permutation and Combinations: Art of Counting

Note that the following 6 arrangements are equivalent:

That is, if each of the women is shifted by a seat in any direction, the seating arrangement remains exactly the
same. That is why we have only 2 arrangements, as shown in the previous figure.
Now that weve done this, the 3 men can be seated in the remaining seats in 3! or 6 ways. Note that we havent
used the formula for circular arrangements now. This is so because, after the women are seated, shifting the
each of the men by 2 seats, will give a different arrangement. After fixing the position of the women (same as
numbering the seats), the arrangement on the remaining seats is equivalent to a linear arrangement.
Therefore the total number of ways in this case will be 2! X 3! = 12.
I hope that you now have some idea about circular arrangements. Next, Ill talk about Combinations.
The previous lessons talked about arrangements or permutations of objects. This lesson will deal with selections
or combinations of objects. In simple words, a combination is a selection made with no regard to order of the
selected objects.
To be a little more clear, lets take an example. Suppose you have 4 different colors

of which you have to mix 2 (in equal proportions) to make a new color. How many new colors can be made?
Notice that mixing Blue and Red is exactly the same as mixing Red and Blue, i.e. the order does not matter.
Lets list all the possible cases.

MathsExplorer.com Mathsclub.org where Mathematics is more colourful

P a g e | 21

Permutation and Combinations: Art of Counting

As you can see there are 6 new colors Black-Blue, Black-Red, Black-Yellow, Blue-Red, Blue-Yellow, RedYellow.
Okay, so how do we count this in a logical way?
Lets first state the problem in a more standard manner: In how many ways can a selection of 2 objects be
made from 4 distinct objects? (say A, B, C and D)
Let the number of combinations be N.
Note that every selection of two particular objects (say A and B) corresponds to 2! different arrangements (AB
and BA). That is, there are 2! times as many permutations as there are combinations (for the case of selecting 2
objects).
But weve already calculated the number of permutations of 4 objects taken 2 at a time. This
equals 4P2. Therefore, according to our logic, 4P2 = 2! X N, or N = 4P2/2! = 6. Makes sense?
What if we had to select 3 objects out of 5? In this case the number of permutations will be 3! or 6 times as the
number of combinations. (The 6 permutations ABC ACB BAC BCA CAB CBA correspond to the selection {A, B,
C}). Therefore the number of combinations will be the number of permutations divided by 3!, which
equals 5P3/3! = 5!/(2!3!) = 10. (These 10 selections are {A,B,C}, {A,B,D},{A,B,E},{A,C,D},{A,C,E},{A,D,E},
{B,C,D},{B,C,E},{B,D,E},{C,D,E})
We can now generalize this expression for the case of selection of r objects out of n distinct objects (0 r n).
The number of selections or combinations will be nPr / r! or n!(nr)!r!. This expression is denoted as nCr or C(n,
r) or (nr). (Well stick with the first one)
Note that each object can be selected exactly once. Later well come to the case when we have to select r
objects out of a lot n distinct types of objects (each available in unlimited supply), such that an object of a given
type can be selected more than once.
To summarize: The number of ways of selecting r objects out of n distinct objects is given by nCr, which
equals n!(nr)!r!
Before moving on to examples, Ill show you one more way to arrive at the same expression for selections.

MathsExplorer.com Mathsclub.org where Mathematics is more colourful

P a g e | 22

Permutation and Combinations: Art of Counting


Consider 6 different objects: A, B, C, D, E and F
Now suppose we have to select 1 object from the lot. Clearly, we have 5 different possible selections: A, B, C, D,
E or F. This is same as 6C1 as we derived in the previous lesson.
Okay, what if we have to select 2 objects? First well select any 1 object in 6C1 ways. Now there are 5 objects
remaining, of which we have to select 1 more. This can be done in 5 ways. So the total number of ways to
complete the selection will be 6C1 x 5.
But there is a problem. In the above way of calculating, weve counted selecting A followed by B and selecting
B followed by A as different. But they are the same, as the order of selection does not matter. Weve counted
the same selection twice.
Therefore to arrive at the correct answer well have to divide the previous answer by 2. Therefore the total
number of selections will be (6C1 x 5)/2.
This is same as 6!5!1!52=6!4!2!=6C2.
What about 3 objects? First, select any 2: 6C2 ways. Then out of the remaining 4, select any 1: 4 ways. Now
again if we multiply 6C2 by 4, we would have counted each selection thrice (A,B followed by C; A,C followed by
B; B,C followed by A). So well have to divide the product by 3: (6C24)/3=6!2!4!43=6!3!3!=6C3. Same answer
as we derived earlier!
I think now we can generalize this logic to n objects, inductively. If we know the way to select r-1 objects the
number of way to select r objects will be obtained by multiplying the previous by (n (r 1)) (as there will be
these many objects left after weve selected r 1, and weve to select 1 more) and then dividing by r (as each
selection will be repeated r times, as we saw above)
The number of ways to select 1 object = n or nC1.
The number of ways to select 2 objects will be nC1n12=n!1!(n1)!n12=n!2!(n2)!=nC2
The number of ways to select 3 objects will be nC2n23=n!2!(n2)!n23=n!3!(n3)!=nC3
The number of ways to select 4 objects will be nC3n34=n!3!(n3)!n34=n!4!(n4)!=nC4
And so on I hope you get the idea.
Weve reached the same expression/formula nCr to obtain the number of selections of r objects from n
distinct objects, using a different logic. (There might be many more.)
Now youll learn a result about binomial coefficients: nCr=nCnr, using the formula as well as by using
combinations.
For example 7C2 equals 7C72 or 7C5.
Lets calculate to be sure.
7C2=7!2!(72)!=7.6.5.4.3.2.1(2.1)(5.4.3.2.1)=21 and 7C5=7!5!(75)!=7.6.5.4.3.2.1(5.4.3.2.1)(2.1)=21
Observe that the 2! and the 5! just got exchanged, while the numerator remains the same. Can you see why
this will hold true for all kinds of nCrs in general ?
If not, heres why. Using the formula for nCr, which equals n!r!(nr)!, we have nCnr=n!(nr)!(n(nr))!=n!(nr)!
(r)!=n!r!(nr)!=nCr
In other words, the r! and (n-r)! get interchanged, and the expressions value remains the same.
Okay, now theres another way we can reach the same result. Recall that nCr is the number of ways to select r
objects from n different objects.
To prove that nCr=nCnr, we have to argue that the number of ways to select r objects from n different objects
should be the same as the number of ways to select n r objects from n different objects.
As an example, suppose we are to select 3 letters from out of the 4 letters A,B,C and D. To select the letters, we
line up the 4 letters, and make the selections by circling out any 3 of these 4 letters. Here are the possible
selections..

MathsExplorer.com Mathsclub.org where Mathematics is more colourful

P a g e | 23

Permutation and Combinations: Art of Counting

There are 4 possible selections (which is same as 4C3, as derived earlier). Now to make the same selections,
we could do the following instead..

That is, put a cross mark on that letter, which is not to be selected. And in how many ways can we do that?
In 4C1 ways! Why? Because out of 4 letters you have to select 1 (and put a cross on it).
So the selection of 3 letters out of 4 can be done by putting a circle each on 3 of the letters to be selected OR by
putting a cross on the letter not to be selected. That is, selecting 3 objects (out of 4) is exactly the same as
selecting 1 object (and throwing it away, thereby selecting the remaining 3 objects).
Well? This means that 4C3 must equal 4C1, as were essentially doing the same thing in both these methods
(which should make the number of ways of both the methods equal)
To generalize, selecting r things out of n different things is exactly same as selecting n r things and removing
them from the lot, so the remaining r things get automatically selected. Therefore, the number of ways to the
former, i.e. nCr must equal the number of ways to do the latter nCnr
And phew ! Were done ! nCr=nCnr
To summarise: Suppose n persons (a1, a2, a3,,an) are to be arranged around a circular table. There are n!
ways in which they can be arranged in a row. On the other hand, all the linear arrangements depicted by
a1, a2, a3, ..., an
an, a1, a2,.,an 1
an 1, an, a1, a2..an 2

MathsExplorer.com Mathsclub.org where Mathematics is more colourful

P a g e | 24

Permutation and Combinations: Art of Counting


.
.
.
a2, a3, a4,.,a1
will lead to the same arrangement for a circular table. Hence each circular arrangement corresponds to n linear
n!
n
arrangements (i.e. in a row). Hence the total number of circular arrangements of n persons is
= (n 1)!.
In other words, the arrangement (permutation) in a row has a beginning and an end, but there is nothing like
beginning or end in circular permutation. Thus, in circular permutation, we consider one object as fixed and the
remaining objects can be arranged in (n 1)! ways (as in the case of arrangement in a row).

Distinction between clockwise and anti-clockwise Arrangements:


Consider the following circular arrangements:
A1
A1
In figure I, the order is clockwise whereas in figure II, the
order is anti-clock wise. These are two different
A3
A2
A2
A3
arrangements. When distinction is made between the
clockwise and the anti-clockwise arrangements of n
II
I
different objects around a circle, then the number of
arrangements = (n 1)!
But if no distinction is made between the clockwise and the anti-clockwise arrangements of n different objects
1
2
around a circle, then the number of arrangements is (n 1)!

For an example, consider the arrangements of beads (all different) on a necklace as shown in figures A and B.
Look at (A) having 3 beads x1, x2, x3 as shown. Flip (A) over
on its right. We get (B) at once. However, (A) and (B) are
really the outcomes of one arrangement but are counted as
two different arrangements in our calculation. To nullify this
redundancy, the actual number of different arrangements is
(n-1)!/2.

x1

x1

x3

x2
A

x3

x2
B

Remarks:

When the positions are numbered, circular arrangement is treated as a linear arrangement.
In a linear arrangement, it does not make difference whether the positions are numbered or not.

Example:

Consider 23 different coloured beads in a necklace. In how many ways can the beads be
placed in the necklace so that 3 specific beads always remain together?

Solution:

By theory, let us consider 3 beads as one. Hence we have, in effect, 21 beads,


number of arrangements = (n-1)! = 20!

'n' = 21. The

MathsExplorer.com Mathsclub.org where Mathematics is more colourful

P a g e | 25

Permutation and Combinations: Art of Counting


Also, the number of ways in which 3 beads can be arranged between themselves is 3! = 3 x 2 x 1
= 6.
Thus the total number of arrangements = (1/2). 20!. 3!.
Example:

In how many ways 10 boys and 5 girls can sit around a circular table so that no two girls sit
together.

Solution:

10 boys can be seated in a circle in 9! ways.


There are 10 spaces inbetween the boys,
which can be occupied by 5 girls in 10p5 ways.
Hence total number of ways

= 9! 10p5 =

9!10!
5!
.

stands for boys)

Example: A question paper consists of 10 questions of which a student needs to answer any 7. In how many
ways can the student make his selection?
Solution: This is a simple case of selection of 7 objects (questions) out of 10 distinct objects. The number of
ways will be 10C7 = 120
Example: Find the number of ways in which a team of 4 can be selected from a group of 10 people A1, A2, A3,
, A10 such that
(i) there are no restrictions
(ii) A1 and A3 must be selected
(iii) A2 must not be selected
(iv) if A6 is selected then A4 must also be selected
(v) if A5 is selected then A7 must not be selected
(vi) A9 is selected if and only if a A10 is selected
Solution: Lets take em one by one!
(i) This ones plain: 10C4
(ii) In this case, we must select A1 and A3. Done. Now, out of the remaining 8, weve to select 2 more. The
number of selections will be 8C2

(iii) In this case, well remove A2 from the group, and select any 4 from the remaining group of 9 people
in 9C4 ways.

(iv) This one is a little complicated. Well have to divide our counting into 2 cases. First in which A6 is selected,
and the second in which A6 is not selected.

MathsExplorer.com Mathsclub.org where Mathematics is more colourful

P a g e | 26

Permutation and Combinations: Art of Counting


Case 1: According to the restriction, if A6 is selected then A4 also must be selected. So well select these 2, plus
2 more from the remaining 8, in 8C2ways.
Case 2: If A6 is not selected, then we can select any 4 out of the remaining 9, in 9C4 ways.
Since it will be either Case 1 or Case 2, the number of ways in both these cases will be added (the addition
principle). The total possible selections will be 8C2 + 9C4.

(v) Again, well divide our counting into 2 cases. First in which A5 is selected, and the second in which A5 is not
selected.
Case 1: According to the restriction, if A5 is selected then A7 must not be selected. So well remove A7, and
select 3 more from the remaining 8: 8C3ways.
Case 2: If A5 is not selected, then we can select any 4 out of the remaining 9, in 9C4 ways.
The total possible selections will be 8C3 + 9C4.

(vi) In this case, either both A9 and A10 will be selected, or both will not be selected.
Case 1: Select A9 and A10, and select 2 more from the remaining 8: 8C2 ways
Case 2: Remove A9 and A10 from the group, and select 4 from the remaining 8: 8C4ways
Total number of ways: 8C2 + 8C4

MathsExplorer.com Mathsclub.org where Mathematics is more colourful

P a g e | 27

Permutation and Combinations: Art of Counting

Lets now complicate things a bit more..


Example: In how many ways can a committee of 8 be selected from a group of 10 men and 12 women, such
that, in the committee,
(i) there are 3 men and 5 women;
(ii) there are atleast 6 women;
(iii) there is atleast one man;
(iv) there are atmost 2 women;
(v) there are more men than women ?
Solution: In this problem, well divide our task of selection into 2 subtasks (a) select the men and (b) select
the women. Since both of these need to be completed to complete the selection, well multiply the number of
ways obtained in both these cases (the multiplication principle)
(i) The men can be selected in 10C3 ways, and the women can be selected in 12C5 ways. Therefore the total
number of ways will be 10C3 x 12C5.
(ii) Now in this case, there can either be 6 women (and 2 men), or 7 women or all 8 women.
Well therefore count the three cases separately and add them together. Similar to (i) above, when there are 6
women (and 2 men), the number of cases will be 10C2 X 12C6. And when there are 7 women (and 1 man) the
number of cases will be 10C1 x 12C7. Finally in the case of 8 women and 0 men, the number of selections will
be 10C0 x 12C8, or simply 12C8. The final answer becomes 10C2 x 12C6 + 10C1 x 12C7 + 12C8
(iii) This could be counted in a similar manner as (ii) above. But there is a better method. If from all the possible
selections, we remove those in which no man is selected (or all-woman committees), well be left with those
cases in which there is atleast one man. Sounds right?
The total number of 8-member committees will be 22C8. The total number of all-women committees will be 12C8.
Therefore our required answer will be 22C8 12C8. Pretty neat!
(iv) Either all men, or 7 men and 1 woman, or 6 men and 2 women: 10C8 + 10C7 x 12C1 + 10C6 x 12C2

Combinations: More Examples


. Lets begin with few more examples...
Q1. There are 10 points in a plane, no 3 of which are collinear. How many different lines can be formed by
joining these points?
Soln. To form a line, all we need to do is select any 2 points (subtask 1), and then join them (subtask 2). The
number of ways to select any points (out of 10 distinct points) will be 10C2. Once we select the points, there is

MathsExplorer.com Mathsclub.org where Mathematics is more colourful

P a g e | 28

Permutation and Combinations: Art of Counting


only 1 straight line which will be formed using these points. Therefore the number of lines will be 10C2 x 1 or 45.
Heres a crazy figure to illustrate.

Q2. Suppose 4 of the 10 points in Q1 are collinear, and no three of the remaining are collinear. Find the number
of straight lines which can be formed. Also find the number of triangles which can be formed.
Soln. Since 4 particular points are now collinear, there will be only 1 line which will be formed by joining any 2 of
these 4, instead of 4C2 if they had been non collinear. So, weve lost 4C2 or 6 lines, and instead we got only 1.
Therefore, the total number of lines will be 10C2 4C2 + 1 = 40
There is one more way in which we can calculate the number of lines. Lets divide the points in two groups: the
collinear group of 4 points, and the non-collinear group of 6 points.
To count the number of lines, we have three possible cases. First, the lines formed using the points of the
collinear group only 1 line. Second, the lines formed using only the points of the non-collinear group 6C2 or
15 lines. Third, select one point from the collinear group (4C1 ways) and the other from the non-collinear group
(6C1 ways) the number of such lines will be 4C1 x 6C1 = 24.
Therefore the total number of lines will be 1 + 15 + 24 = 40. This was a bit longer method, but nevertheless an
important method of counting dividing into subcases and adding the results together. Another crazy figure..

MathsExplorer.com Mathsclub.org where Mathematics is more colourful

P a g e | 29

Permutation and Combinations: Art of Counting

What about the number of triangles?


Suppose all of the points were non-collinear. The number of triangles would have been 10C3, because to form a
triangle, we need to select any three points.
But! There are four culprit points, which wont form a triangle, as they are collinear. Weve lost 4C3 triangles,
which would have been formed, had these 4 been non-collinear.
Therefore, the total number of triangles is 10C3 4C3.
Try using the second method (of making cases) and see if you arrive at the same answer.

Q3. Consider a card game, in which each player is dealt 5 cards. In how many ways can a player obtain the
following combinations?
A deck of cards contains 52 cards with 4 different suits, and 13 kinds of cards in each suit, as shown below.
A 2 3 4 5 6 7 8 9 10 J Q K
A 2 3 4 5 6 7 8 9 10 J Q K
A 2 3 4 5 6 7 8 9 10 J Q K
A 2 3 4 5 6 7 8 9 10 J Q K
(i) All cards of the same suit:

MathsExplorer.com Mathsclub.org where Mathematics is more colourful

P a g e | 30

Permutation and Combinations: Art of Counting


3 7 8 9 K
(ii) Two cards of the same kind and the rest three of the same kind:
4 4 K K K
(iii) Two cards of the same kind and rest all different:
4 4 K 9 7
(iv) Cards in sequence:
4 5 6 7 8
Soln. Note that the order of the cards in a players hand is irrelevant. We are only interested in the combination
of cards.
(i) If we need 5 cards of the same suit, well first have to select a suit, in 4C1 ways. We now have a suit of 13
different cards, of which 5 cards need to be selected. This can be done in 13C5 ways. Therefore the total number
of ways will be 4C1 x 13C5 = 5148
(ii) Well divide this task into the following subtasks select the kind to be paired, then select 2 cars of that kind.
Then select the kind to be tripled, then select 3 cards of that kind.
For the pair we first select the kind in 13C1 ways, and then select any two cards of that kind in 4C2 ways.
We are now left with 48 cards, 12 kinds of cards of 4 different suits. Now well select another kind in 12C1 ways
and select 3 cards of that kind in 4C3ways.
The total number of combinations is therefore 13C1 x 4C2 x 12C1 x 4C3 = 3744
(iii) Similar to the previous part we can select the pair in 13C1 x 4C2 ways. But now we have to be careful. Of the
48 cards remaining, we cannot select any 3 cards, because it might happen they all are of the same kind.
To handle this, well first select 3 different kinds, in 12C3 ways, say we get 5, 8 and K. Now each of these cards
can be of any of the 4 suits. There are 4 possibilities for the 5, 4 for the 8 and 4 for the K. That means, for a
given selection of 3 kinds of cards, there are 4 x 4 x 4 or 64 different combinations.
Therefore, the total number of combinations of the remaining 3 cards will be 12C3 x 64, making the answer equal
to 13C1 x 4C2 x 12C3 x 64 = 1098240
(iv) In this case well first select the sequence to be formed. There are 9 possibilities: (A,2,3,4,5), (2,3,4,5,6)
(9, 10, J, Q, K). Similar to the previous part, there will be 4 possibilities for each of the 5 cards selected.
Therefore the total number of combinations will be 9 x 4 x 4 x 4 x 4 x 4 = 147456
Perplexed? Me too! Try a few more cases yourself..
(v) Two pairs, and the fifth card different
4 4 5 5 K

MathsExplorer.com Mathsclub.org where Mathematics is more colourful

P a g e | 31

Permutation and Combinations: Art of Counting


(vi) Three of the same kind, and the rest two different
7 7 7 4 K
(vii) Four of the same kind
4 4 4 4 K

MathsExplorer.com Mathsclub.org where Mathematics is more colourful

P a g e | 32

Permutation and Combinations: Art of Counting

Permutations involving Identical Objects 33


So far weve talked about permutations of objects which were distinct. Things change when some (or all) of the
objects to be arranged are identical.
Suppose we have to arrange 5 balls in a row of which 3 are of the same color, and the other two are of different
colors. How many different arrangements are possible?
Heres one way to think about this problem.
Lets designate five boxes, each of which is to be filled by one ball.

First, well place the identical balls in any three of the five boxes. This can be done in 5C3 ways. Heres one of
them.

Next, in the remaining two boxes, well place the remaining two balls. This can be done in 2! ways.

or

Therefore the total number of arrangements will be 5C3 x 2! = 5!/3! = 20.


Now lets say 3 of the 5 balls are of the same colour and the other 2 are of different colours. How many different
arrangements are possible?
MathsExplorer.com Mathsclub.org where Mathematics is more colourful

P a g e | 33

Permutation and Combinations: Art of Counting


Similar to the method used above, well first place the identical balls in any 2 of the 5 boxes, in 5C2 ways. One of
the arrangement is given below.

Now in the remaining three boxes, the remaining three balls can be placed in 3! ways.

Therefore the total number of arrangements will be 5C2 x 3! = 5!/2! = 60.


We could also have placed the different coloured balls first. This can be done by first selecting any three boxes,
(in 5C3 ways), and then placing the balls in the boxes (3! ways).
After weve placed these three balls, the remaining two balls will be filled with the two identical balls (which can
be done in only one way).
The answer remains the same (5C3 x 3!) x 1 = 5!/2! = 60
Do you see a pattern? Had the 5 balls been all of different colours, the total number of arrangements would have
been 5!. When 3 of the balls were identical, the answer changed to 5!/3!, and when 2 of them were identical, the
answer was 5!/2!. Can we arrive at a formula or a generalization?
Suppose there are n objects, to be arranged in a row, of which p objects are identical, and the remaining are
distinct. How many arrangements are possible?
In this case we have n boxes, each to be filled with one ball. Well first select any p boxes out of the n boxes,
in nCp ways. Now well place the identical balls in the selected boxes, one in each. Were now left with n p
boxes and n p balls, which can be placed in the boxes in (n p)! ways.
Therefore the total number of ways is nCp (n p)! or n!/p!

Permutations & Combinations Permutations involving Identical Objects: Examples


This lesson will cover a few examples to illustrate what we learnt in the previous lesson.
Q1. Find the number of arrangements (in a row) of the letters of the following words:
(i) TWITTER
(ii) GOOGLE

MathsExplorer.com Mathsclub.org where Mathematics is more colourful

P a g e | 34

Permutation and Combinations: Art of Counting


Sol. (i) There are a total of 7 letters to be arranged in a row of which 3 Ts are identical (and the remaining
different). The total number of permutations will be 7!/3! or 840.
(ii) In this case, there are a total of 6 letters which 2 Gs are identical and 2 Os are identical. The number of
arrangements will be 6!/2!2! or 180
Q2. Consider the following grid

Here is the situation. You are at the bottom left corner of the grid (the red dot), and are supposed to reach the
top right corner of the grid. But there is a condition you are only allowed to travel along the grid lines, either
one step rightward or one step upward. In how many different ways can you reach the top right corner? One
such way is shown below.

Sol. This is one example of problem where it isnt very obvious whether its based on permutations or
combinations. Youll come across many such problems, which wont involve direct application of any formula. In

MathsExplorer.com Mathsclub.org where Mathematics is more colourful

P a g e | 35

Permutation and Combinations: Art of Counting


such problems, you need to establish a logic for counting first, and accordingly use the basic counting principles
(or formulas, if applicable).
Lets come back to the problem again. Notice that whatever be the path chosen, there will always be a total of 5
steps in the rightward direction and 4 in the upward direction. Here are a few examples.

MathsExplorer.com Mathsclub.org where Mathematics is more colourful

P a g e | 36

Permutation and Combinations: Art of Counting

MathsExplorer.com Mathsclub.org where Mathematics is more colourful

P a g e | 37

Permutation and Combinations: Art of Counting

Permutations & Combinations Combinations involving Identical Objects


This lesson will cover briefly a few simple cases involving selections (or combinations) involving identical
objects.
Suppose you have five identical red balls, of which youve to choose any two. How many different combinations
are possible?
It doesnt matter which two balls you select whether you select the first two, or the last two, or the third and the
fourth youll end up with the same selection each time. Why? Because the balls are identical.
Therefore the number of ways to select two balls will be one.
And if you had to select three balls, how many choices would you have? The answer remains the same one.
And four balls? Again, only one possible selection.
To formalize things a bit
The number of ways to select r objects out of n identical objects equals 1 (where 0 r n)
Okay, that was simple to understand. Lets complicate things a bit now.
Suppose you now have the following lot three red balls and three balls of different colors

If youre to select three out of these, how many different choices do you have?
The answer again not be 6C3 (the case where all of the balls are of different color), because identical balls will
reduce the number of choices, as we saw above.
So how can we count the possible combinations in this case? By making cases
We first separate the balls into two lots the identical balls (say, lot 1) and the distinct balls (lot 2).

MathsExplorer.com Mathsclub.org where Mathematics is more colourful

P a g e | 38

Permutation and Combinations: Art of Counting


Next, we divide our selection into two subtasks select from lot 1 and select from lot 2.
Finally, we make cases..
I. All the three balls from lot 1: 1 way
II. Two from lot 1 and one from lot 2: 1 x 3C1 ways
III. One from lot 1 and two from lot 2: 1 x 3C2 ways
IV. All the balls from lot 2 3C3 ways
.. and we add all the cases: 1 + 3 + 3 + 1 = 8 ways
Hmm... Its getting complicated already.
And you must be thinking, Wait.. what about the case where weve to select 3 balls from a lot of 4 red, 5 green
and 6 different coloured balls?
Exponent of Prime p in n!:
Let

be

prime

number

and

be

1, 2, 3, .., (n 1), n which is divisible by p is

positive
n
p

integer.

p, where

n
p

Then

the

last

integer

denotes the greatest integer less than or

n
p

equal to

.
10
3

For Example,

12
5

15
3

= 3,

= 2,

= 5 etc.

Let Ep (n) denotes the exponent of the prime p in the positive integer n. Then,
Ep (n !) = Ep (1 . 2. 3 ..(n 1) n)

n
p
p

p.2p.3p.......

= Ep

n
p

+ Ep

Remaining integers between 1 and n are not divisible by p]

1.2.3.......

Now, the last integer amongst 1, 2, 3, ..

Ep (n !) =

n
p

n
2
p

n
p

n /p
p p

which is divisible by p is

n
2 p
p

+ Ep


1.2.3....... n
2

amongst

Continuing in this manner, we get

MathsExplorer.com Mathsclub.org where Mathematics is more colourful

P a g e | 39

Permutation and Combinations: Art of Counting

Ep (n !) =
ps + 1

n
p

n
2
p

n
3
p

+ ..+

n
s
p

, where s is the largest positive integer such that ps n <

Example:

Find the exponent of 3 in 100 !

Solution:

Let Ep (n) denotes the exponent of p in n.

Then, Ep (n !) =
that ps n < ps + 1

n
p

n
2
p

Here, n = 100, p = 3

+ ..+

n
s
p

, where s is the largest positive integer such

34 100 < 35 s = 4.

100
3

So, E3 (100 !) =

n
3
p

100
32

100
33

100
34

= 33 + 11 + 3 + 1 = 48
Hence, the exponent of 3 in 100 ! is 48.

Example:

Find the number of zeros at the end of 100 !.

Solution:

In terms of prime factors 100! can be written as 2a3b5c, 7d..


Now,
100 100 100 100 100 100
2 22 23 24 25 26

E2 (100!) =
= 50 + 25 + 12 + 6 + 3 + 1 = 97
100 100
5 52

and E5 (100 !) =

= 20 + 4 = 24.

Therefore,
100 ! = 297 3b 524 7d .= 273 (2 5)24 7d
= 1024 273 3b 7d .

MathsExplorer.com Mathsclub.org where Mathematics is more colourful

P a g e | 40

Permutation and Combinations: Art of Counting


Thus, the number of zeros at the end of 100 ! is 24.

Remarks:
n
k

The number of integers from 1 to n which are divisible by k is


. e.g. the number of integers from 1
100
100
2 50
3

to 100 which are divisible by 2 and 3 individually are


and
= 33 respectively.

The

number

of

natural

numbers

from

to

n,

which

are

perfect

kth

powers,

110
1/k

[n ]. e.g. the number of perfect squares from 1 to 110 is [

] = 10.

MathsExplorer.com Mathsclub.org where Mathematics is more colourful

is

P a g e | 41

Permutation and Combinations: Art of Counting


ASSIGNMENTS (Subjective)
LEVEL I
1.

Find the total number of words which can be formed out of the letters of the word ALLAHABAD such that
the vowels occupy the even positions.

2.

On a railways there are 20 stations. Find the number of different tickets required in order that it may be
possible to travel from every station to every station

3.

A car will hold two persons in the front seat and one in the rear seat. If among six persons only two can
drive. Find the number of ways in which the car can be filled.

4.

The letters of the word CRICKET are written in all possible orders and then are arranged in a dictionary
what is the rank of the word CRICKET.

5(a).

A cricket team of eleven players is to be formed from 16 players including 4 bowlers and 2 wicket keeper.
In how many different ways can a team be formed so as to contain atleast 3 bowlers and atleast one
wicket keeper.

(b).

In how many ways can a team of 3 chemistry teachers and 4 mathematics teachers can be formed from
8 chemistry teachers and 10 mathematics teachers such that a particular chemistry teacher refuses to be
in the team if a particular mathematics is in the team.

6.

A round table conference is to be held between 20 delegates of 20 countries. In how many ways can
they be seated if two particular delegates
(a) always sit together.
(b) never sit together.

7.

(a) In how many ways can 2n people be divided into n pairs ?


(b) In how many ways can you equally distribute 100 packages of food to 10 refugees?
(c) In how many ways mn things be equally distributed in n groups?
(d) In how many ways can 22 books be divided into 5 students so that two students will have 5 books
each and the other three students will have 4 books each.

8.

There are 5 mangoes and 4 apples. In how many different ways can a section of fruits be made if

MathsExplorer.com Mathsclub.org where Mathematics is more colourful

P a g e | 42

Permutation and Combinations: Art of Counting


(a) fruits of the same kind are different.
(b) fruits of the same kind are identical?

9.

Five subjects are to be allotted to 6 different periods. In how many ways can this be done if no period has
to go un allotted?

10.

In how many ways can 3 girls and 9 boys be seated in two vans, each having numbered seats, 3 in the
front and 4 at the back? How many sitting arrangements are possible if 3 girls should sit together in the
back row on adjacent seats?

MathsExplorer.com Mathsclub.org where Mathematics is more colourful

P a g e | 43

Permutation and Combinations: Art of Counting


LEVEL II

1.

How many numbers of 4 digits can be made with the digits 0, 1, 2, 3, 4, 5, which are divisible by 3, digits
being unrepeated in the same number? How many of these will be divisible by 6?

2.

In how many ways can 21 identical white balls and 19 identical black balls be arranged in a row so that
no two black balls may be together? What will be the result if balls are considered to be distinct?

3(a).

A man has seven relatives, four women and three men. His wife also has seven relatives, three women
four men. In how many ways they invite three women and three men so that three of them are the mans
relatives and three his wifes.

(b).

A box contains two white balls, three black balls and four red balls. In how many ways can three balls be
drawn
from
the
box if
at
least
one
black
ball
is
to
be
included
in
the draw.

(c).

Find the total number of seven digit numbers the sum of whose digit is odd.

4.

A person wants to hold as many different parties as he can out of 24 friends, each party consisting of the
same number. How many should he invite at a time ? In how many of these would the same man be
found ?

5.

There are n straight line in a plane, no two of which are parallel and no three pass through the same
1
8
point, their points of intersection are joined. Show that the number of fresh lines thus introduced is n(n
1)(n 2)(n 3).

6(a).

Suppose n different games are to be given to n children. In how many ways can this be done so that
exactly one child gets no game.

(b).

Find the numbers of rectangles excluding square from a rectangle of size 15 10.

7.
In how ways can 10 persons take seats in a row of 24 fixed seats so that no two person take consecutive
seats?

MathsExplorer.com Mathsclub.org where Mathematics is more colourful

P a g e | 44

Permutation and Combinations: Art of Counting

8.

In how many ways can the letters in the English alphabet be arranged, so that there are 7 letters
between the letters a and b.

9(a).

A mathematics paper has 12 questions divided into 3 sections. A, B, C, each having


4 questions. In how many ways can you answer 5 questions selecting at least one question from each
part.

(b).

In an examination, the maximum marks for each of 3 papers is n and for fourth paper 2n. Prove that the
number
of
ways
in
which
a
candidate
can
get
3n
marks
is
1
6
(n + 1) (5n2 + 10n + 6).

10.

From a given number of 4n books, there are three sets of n identical books on physics, chemistry and
mathematics. The remaining n are distinct books on other subjects. Find the number of ways of choosing
n out of 4n books.

ANSWERS (Subjective)
LEVEL I
1.

60

2.

380

3.

40

4.

531

5(a).

2472

(b).

6(a).

2 18!

(b).

17 18!
100 !

2n !
n ! 2n
7(a).

C3 10C4 - 7C2 9C3

10 !10
(b).

(mn)!

22!

3! 2! 5! ( 4! ) 3

(m! ) n !
(c).

(d).

8(a).

29 1

(b).

29

9.

1800

10.

14

P12, 24 11P9

MathsExplorer.com Mathsclub.org where Mathematics is more colourful

P a g e | 45

Permutation and Combinations: Art of Counting

LEVEL II

1.

96, 52

2.

1540, 1540 21! 19!

3(a).

485

(b).

64

(c).

4500000

4.

12 friends,

6(a).

(b).

5940

8.

36 24!

10.

2n 3(n2 + 7n + 8)

C2 n!

23

C11 parties

15 !
5!
7.
9(a).

56

MathsExplorer.com Mathsclub.org where Mathematics is more colourful

P a g e | 46

Permutation and Combinations: Art of Counting


ASSIGNMENTS (Objective)
LEVEL - I
1.

The number of ways in which 6 men can be arranged in a row so that three particular men are
consecutive, is
(A) 4P4

(B) 4P4 3P3

(C) 6P6 3P3

(D) 3P3 3P3

B
2.

Every one of the 10 available lamps can be switched on to illuminate certain hall. The total number of
ways in which the hall can be illuminated is
(A) 55
(B) 1023
(C) 210

(D) 20!

B
5

52 j

C3

j 1

3.

Value of expression 47C4 +

is equal to

(A) 52C4

(B) 52C2

(C) 52C6

(D) none of these

A
4.

A polygon has 65 diagonals. The number of its sides is


(A) 8

(B) 10

(C) 11

(D) 13

D
5.

Set A has 3 elements and set B has 4 elements, the number of injections that can be defined from A to B,
is
(A) 144

(B) 12

(C) 24

(D) 64

C
6.

The letter of the word MATHS are written in all possible orders and these words are written out as in
dictionary, then the rank of the word MATHS is
(A) 53

(B) 54

(C) 55

(D) 56

MathsExplorer.com Mathsclub.org where Mathematics is more colourful

P a g e | 47

Permutation and Combinations: Art of Counting


A
7.

Number of ways of painting the faces of a cube with different colours


(A) 1

(B) 6

(C) 6!

(D) 26

A
8.

The maximum number of different permutations of 4 letters of the word EARTHQUAKE is


(A) 1045

(B) 2190

(C) 4380

(D) 2348

B
9.

Everybody in a room shakes hand with everybody else. The total numbers of handshakes is 66. The total
number of persons in the room is
(A) 11

(B) 12

(C) 13

(D) 14

B
10.

The number of ways of selecting two numbers from the set {1, 212} whose sum is divisible by 3 is
(A) 66

(B) 16

(C) 6

(D) 22

D
11.

The number of 4-digit number that can be made with the digits 1, 2, 3, 4, 5 in which at least two digits are
identical, is
(A) 45 5!

(B) 600

(C) 54 4!

(D) none of these

A
12.

Number of triangles that can be formed joining the angular points of decagon is
(A) 30

(B) 20

(C) 90

(D) 120

D
(a 2 a )

C2 ( a

13.

a)

C4

, then a =
(A) 2

(B) 3

MathsExplorer.com Mathsclub.org where Mathematics is more colourful

P a g e | 48

Permutation and Combinations: Art of Counting


(C) 4

(D) none of these

B
14.

The product of r consecutive integers is divisible by


r 1

k
k 1

(A) r

(B)

(C) r !

(D) none of these

A,B,C
15.

Total numbers of 5-digit telephone numbers that can be composed with distinct digits is
(A) 10P2

(B) 10P5

(C) 10C2

(D) none of these

D
16.

If 7 points out of 12 are in same straight line, then number of triangles formed is
(A) 19

(B) 158

(C) 185

(D) 201

C
17.

The sides AB, BC, CA of a triangle ABC have 3, 4 and 5 interior points respectively on them. The total
number of triangles that can be constructed by using these points as vertices is
(A) 220

(B) 204

(C) 205

(D) 195

C
18.

How many numbers between 5000 and 10,000 can be formed using the digits 1, 2, 3, 4, 5, 6, 7, 8, 9
when each digit is appearing not more than once in each number?
(A) 58P3

(B) 58C3

(C) 5!8P3

(D) 5!8C3

A
19.

The number of all the odd divisor of 3600 is


(A) 45

(B) 4

(C) 18

(D) 9

D
20.

The number of 5-digit numbers in which no two consecutive digits are identical, is

MathsExplorer.com Mathsclub.org where Mathematics is more colourful

P a g e | 49

Permutation and Combinations: Art of Counting


(A) 92 83

(B) 9 84

(C) 95

(D) none of these.

C
21.

22.

The number of even proper divisors of 1008 is


(A) 23

(B) 21

(C) 20

(D) none of these.

The total number of ways in which a begger can be given at least one rupee from four
25-paise coins, three 50 paise coins and 2 one-rupee coins is
(A) 54
(B) 50
(C) 52

23.

24.

25.

26.

(D) none of these.

The number of positive integers with the property that they can be expressed as the sum of the cubes of
2 positive integers in two different ways is
(A) 10

(B) 100

(C) 40

(D) .

In an examination, the maximum marks for each of three papers are 50 each. Maximum marks for the
fourth paper are 100. The number of ways in which the candidate can score 60% marks in the aggregate
is
(A) 110556

(B) 110500

(C) 110356

(D) none of these.

India and Sri Lanka play one day international seties until one team wins 4 matches. No match ends in a
draw. The number of ways in which India can win the series is
(A) 35

(B) 70

(C) 40

(D) none of these.

The number of rectangles excluding squares from a rectangle of size 9 6 is


(A) 789

(B) 790

(C) 791

(D) none of these.

MathsExplorer.com Mathsclub.org where Mathematics is more colourful

P a g e | 50

Permutation and Combinations: Art of Counting


27.

Eleven scientists are working on a secret project. They wish to lock up the documents in a cabinet such
that the cabinet can be opened if and only if six or more scientists are present; then the least number of
locks needed is
(A) 1
(B) 11
(C) 252
(D) 462

28.

All the students of a class send New year greetings to one another. If the postman delivers 1190
greeting cards to the students of the class. Then the number of students in it is
(A) 15
(B) 25
(C) 35

29.

30.

31.

(D) 45

The number of permutations of 4 letter word taken from the word EXAMINATION having two alike and
two different letters is
(A) 752

(B) 756

(C) 758

(D) 780

The total number of words that can be formed of the word MATHEMATICS that begin with T and also
end with T is
(A) 90720

(B) 907200

(C) 10080

(C) 91780

The number of ways of distributing 30 balls among three persons so that one person to get 10 balls ,
another persons 15 and 5.

30!
10!15! 5!

30!
10!15! 5!

(A)

(B)

.3!

30!
10!15! 5!
(C)

.3

(D) None of these

C
32.

33.

The number of proper divisors of 1008 is


(a) 23

(b) 28

(c) 20

(d) 24

In an examination there are three multiple choice questions (one option correct only) and each question
has 4 choices. Number of ways in ways in which a student can fail to get all answer correct is

MathsExplorer.com Mathsclub.org where Mathematics is more colourful

P a g e | 51

Permutation and Combinations: Art of Counting

34.

35.

36.

37.

(A) 64

(B) 63

(C) 65

(D) None of these

Let Tn denotes the number of triangles which can be formed using the vertices of a regular polygon of n
sides. If Tn+1 Tn = 21, then n equals
(a) 5

(b) 7

(c) 6

(d) 4

The number of words which can be made out of the letter of the word MOBILE when consonants
always occupy odd places is
(A) 20

(B) 36

(B) 30

(D) 720

If 5 parallel straight lines are intersected by 4 parallel straight line, then the number of parallelogram
formed is
(A) 60

(B) 207

(C) 101

(D) 126

The figures 4 , 5 , 6 , 7, 8 are written in every possible order. The number of numbers greater than 56000
is
(A) 72

(B) 90

(C) 96

(D) 98

38.

The no. of arrangements of the letters of the word BANANA in which the two Ns do not appear
adjacently is
(a) 40
(b) 60
(c) 80
(d) 100

39.

The number of ways of selecting 2 white squares on a normal chess board, such that they dont lie on
the same row or column is equal to

40.

(A) 96

(B) 400

(C) 480

(D) 496

Number of ways in which 6 persons can be seated around a table so that two particular persons are
never seated together is equal to
(a) 144
(b) 72

MathsExplorer.com Mathsclub.org where Mathematics is more colourful

P a g e | 52

Permutation and Combinations: Art of Counting


(c) 36

41.

(d) 240

Two players P1 and P2 plays a series of 2n games. Each game can result in either a win or loss for P 1,
total numbers of ways in which P1 can win the series of these games is equal to

1
2
(A)

(22n 2nCn)

1
2
(C)

42.

(2n 2nCn)

44.

(B)

2n

(2n 2

2n

1
2
(D)

Cn)

Cn )

(D) 45

If the (n + 1) numbers a, b, c, dbe all different and each of them a prime number, then the number of
different factors (other than 1) of am. b. c. d..is
(A) (m + 1) 2n

(B) (m + 1) 2n 1

(C) (m + 1) 2n + 1

(D) none of these

Give that n is odd, the number of ways in which three numbers in A.P. can be selected from 1, 2, 3, ..,
n is
(n 1)2
4

(A)

(n 1)2
4

(B)
(n 1)2
2

(C)

45.

(22n 2

All the students of a class send New Year greetings to one another. If the postman delivers 1190
greeting cards to the students of the class. Then the number of students in it is
(A) 15
(B) 25
(C) 35

43.

1
2

(n 1)2
2

(D)

The number of rectangles that you can find on a chess board is


(A) 144

(B) 1296

(C) 256

(D) none of these

MathsExplorer.com Mathsclub.org where Mathematics is more colourful

P a g e | 53

Permutation and Combinations: Art of Counting


46.

The letters of the world longest are arranged at random. The probability that the vowels may occupy
only odd positions, is
(A) 2/7

(B) 5/7

(C) 1/7

(D) 3/7

A
47.

The number of arrangement of the letters of the word Pencil so that e always comes earlier then i , is
(A) 290

(B) 340

(C) 360

(D) 320

C
48.

The no. of words which can be made from the letters of the word BANANA is
(A) 60

(B) 120

(C) 360

(D) 720

A
49.

The number of permutations of all the letters of the word EXCERCISES is


(A)60480
(B) 30240
(C)151200
(D) 56504

C
50.

There are 10 books and 3 copies of each. The number of ways in which a selection can be made from
them.
(A) 103

(B) 310

(C) 410 1

(D) 114 1

B
51.

The number of straight lines that can be formed by joining 20 points of which 4 points are collinear is
(A) 183

(B) 185

(C) 186

(D) 190

52.

Number of ways in which 6 persons can be seated around a table so that two particular persons are
never seated together is equal to
(a) 144
(b) 72
(c) 36
(d) 240

53.

The number of ways in which 5 women can draw water from 5 taps, no tap remaining unused is
(A) 52

(B) 5!

MathsExplorer.com Mathsclub.org where Mathematics is more colourful

P a g e | 54

Permutation and Combinations: Art of Counting


(C) 55

54.

(D) None of these

The number of all the odd divisors of 3600 is


(A) 45

(B) 4

(C) 18

(D) 9

55.

The number of 5 digit numbers that can be formed by using 1 and 2 is


(A) 5!
(B) 25
5
(C) C2
(D) none of these

56.

The maximum number of points into which 4 circles and 4 straight lines intersect is
(A) 26

(B) 50

(C) 56

(D) 72

B
57.

In how many ways the number 10800 be resolved as a product of two factors (A) 35
(B) 36
(C) 29
(D) 30

D
58.

Number of ways in which the letters of the word MATHEMATICS can be arranged without changing the
order of vowels is

4! 7!

(11)!
4! (2! ) 3
(A)

(2! ) 3
(B)

11
4 C. 7!
2

11!
(2! ) 3

(2! )

(C)

(D)

C
59.

Number of nine digit numbers, which can be written down using the first nine natural numbers without
repetition, If the digits 2 & 5 are never side by side is
(A) 2(8!)
(B) 8.(8!)
(C) 7. 8!
(D) 6. 8!

C
60.

The results of 11 chess matches (as, win, lose or draw) are to be forcast. Out of all possible forcasts, the
number of ways in which 8 correct & 3 incorrect results can be forcast is :

MathsExplorer.com Mathsclub.org where Mathematics is more colourful

P a g e | 55

Permutation and Combinations: Art of Counting


(A) 8! 3!

11
3 C 81

(B)

11
8 C8

(C)

(D) none

C
61.

The number of ways in which 5 beads, chosen from 8 different beads be threaded on to the ring is
(A) 672
(B) 1344
(C) 336
(D) none

A
62.

The total of words that can be formed out of the word MATHEMATICS that begin with A and also end
with A is
11!
(2! ) 3

(A)

11!
(2! ) 2

(B)
9!
(2! ) 2

(C)

9!
(2! ) 2

(D)

C
63.

The number of ways in which Rs 20 can be distributed among five persons 50 that each person receives
at least Rs 3 is
(A) 63

(B) 126

(C) 252

(D) none of these.

64.

A five digit number divisible by 3 is to be formed using the numbers 0, 1, 2, 3, 4, 5 without repetition total
number of ways in which the number can be formed is
(A) 216

(B) 240

(C) 600

(D) 3125.

A
65.

Six boys and six girls sit along a line alternately in x ways; and along a circle (again alternately) in y
ways, then
(A) y = 10 x

(B) y = 12 x

(C) x = 10y

(D) x = 12y.

MathsExplorer.com Mathsclub.org where Mathematics is more colourful

P a g e | 56

Permutation and Combinations: Art of Counting


66.

The number of ways in which three distinct numbering A.P can be selected from the set
{1, 2, 3 .12} is equal to
(A) 18

(B) 24

(C) 30

(D) 15.

C
67.

The number of triangle whose vertices are at the vertices of an octagon but none of whose side happen
to come from the side of the octagon is
(A) 16

(B) 15

(C) 54

(D) 27.

A
68.

The total number of words that can be formed of the word MATHEMATICS that begin with T and also
end with T is
(A) 90720

(B) 907200

(C) 10080

(D) 91780

A
69.

In an examination a minimum is to be secured in each of 5 subjects for a pass. The number of ways in
which a student can fail is
(A) 31

(B) 36

(C) 42

(D) None of these

70.

The number of ways in which N positive signs and n negative sign (Nn) may be placed in a row so that
no two negative signs are together is
(A) NCn

(B) N+1Cn

(C) N!

(D) N+1Pn

71.

The number of ways of selecting 8 letters from 24 letters of which 8 are a, 8 are b and the rest unlike is
given by
(A) 27

(B) 828

(C) 10 27

(D) None of these

MathsExplorer.com Mathsclub.org where Mathematics is more colourful

P a g e | 57

Permutation and Combinations: Art of Counting


72.

Number of ways in which 6 persons can be seated around a circular table so that two particular persons
are never seated together is equal to
(A) 480

(B) 72

(C) 120

(D) 240

B
73.

The number of diagonals of hexagon is


(A) 3

(B) 6

(C) 9

(D) 12

C
74.

The number of 10 digits that can be written by using the digits 1 and 2 is
(A) 1010

(B) 10P2

(C) 210

(D) 10!

C
75.

The number of all the odd divisors of 3600 is


(A) 45

(B) 4

(C) 18

(D) 9

MathsExplorer.com Mathsclub.org where Mathematics is more colourful

P a g e | 58

Permutation and Combinations: Art of Counting


LEVEL - II

1.

The number of ways in which N positive signs and n negative signs (N n) may be placed in a row so
that no two negative signs are together is
(A) NCn

(B) N+1Cn

(C) N!

(D) N+1Pn

B
2.

m men and n women to be sited in a row so that no two women sit together if m > n, then the number of
ways in which they can be sited is

m! (m 1)!
m n 1 !

m! n!
m n !
(A)

(B)

m! n!
m n 1 !
(C)

(D) none of these

B
3.

A parallelogram cut by n two sets of m lines parallel to its sides. The number of parallelogram thus
formed is
(A) (mC2)2

(B) (m + 1C2)2

(C) (m + 2C2)2

(D) none of these

C
4.

Number of permutations of n different objects taken r ( 3) at a time which includes 3 particular objects,
is
(A) nPr 3!

(B) nPr3 3!

(C) n3Pr3

(D) rP3 n3Pr3

D
5.

The number of words from the letters of the word BHARAT in which B and H never come together is
(A) 360

(B) 240

(C) 120

(D) none of these

B
6.

Cr-3 + 3nCr-2 + 3nCr-1 + nCr is equal to

(A) n+2Cr-1

(B)

n+2

Cr

MathsExplorer.com Mathsclub.org where Mathematics is more colourful

P a g e | 59

Permutation and Combinations: Art of Counting


(C) n+2Cr+1

(D) n+3Cr

D
7.

Number of ways 6 different flowers can be given to 10 girls, if each can receive any number of flowers is
(A) 610

(B) 106

(C) 60

(D) 10C6

Cn

8.

1
1
6
Cn
Cn
, then the value of n is

(A) 3

(B) 5

(C) 2

(D) 15

C
9.

Number of ways in which 5 boys and 5 girls can form a circle such that boys and girls alternate is
(A) 2180

(B) 55

(C) 2880

(D) 2675

C
10.

The least positive integral value of x which satisfy the inequality 10Cx 1 > 2.10Cx is
(A) 5

(B) 6

(C) 7

(D) 8

D
11.

The number of ways in which 5 colour beads can be used to form a necklace, is
(A) 15

(B) 20

(C) 12

(D) 10

C
12.

The number of different signals which can be given from 6 flags of different colours taking one or more at
a time
(A) 1958

(B) 1956

(C) 16

(D) 64

B
13.

Sum of the digits in the unit place of all the numbers formed with the help of 3, 4, 5, 6 taken all at a time
is

MathsExplorer.com Mathsclub.org where Mathematics is more colourful

P a g e | 60

Permutation and Combinations: Art of Counting


(A) 432

(B) 108

(C) 36

(D) 18

B
14.

Number of ways in which 9 identical balls can be placed in three identical boxes is
9!

3 ! 4
(A) 55

(B)

9!

3 ! 3
(C)

(D) 12

D
15.

Number of words that can be formed out of the letters of the word COMMITTEE is
9!

2 !

9!

2! 2

(A)

(B)

9!
2!
(C)

(D) 9!

A
16.

Number of six letters words that can be formed using the letters of word ASSIST in which Ss alternate
with other letter is
(A) 12

(B) 24

(C) 18

(D) none of these

B
17.

The number of ways in which ten candidate A1, A2, .., A10 can be ranked such that A1 is always above
A10, is

10 !
2
(A) 10!

(B)

(C) 2 10!

(D) none of these

B
18.

The number of ordered triplets of positive integers which are solutions of the equation
is
(A) 5081

x + y + z = 100

(B) 6005

MathsExplorer.com Mathsclub.org where Mathematics is more colourful

P a g e | 61

Permutation and Combinations: Art of Counting


(C) 4851

(D) none of these.

C
19.

The number of zeros at the end of (127)! is


(A) 31

(B) 30

(C) 0

(D) 10

A
20.

B
21.

The number of permutations of the letters a, b, c, d, e, f, g such that neither the pattern beg nor the
pattern cad appears is
(A) 4106

(B) 4806

(C) 4776

(D) 5120

The number of ways in which 5 woman can draw water from 5 taps , no tap remaining unused is
(A) 52

(B) 5!

(C) 55

(D) None of these

B
22.

In a college of 300 students , every student reads 5 newspaper and every newspaper is read by 60
students. The number of newspaper is
(A) less than 25

(B) at lost 20

(C) exactly 20

(D) exactly 25

D
23.

24.

Number of triangles which can be formed from 12 points out of which 7 are collinear
(A) 105

(B) 210

(C) 175

(D) 185

The number of n letter words that can be formed using letters of the word AJMER such that it contain A
and E & doesnt contain R.
(A) 4n + 2.3n 2n
(B) 4n + 2.3n + 2n
(C) 4n 2.3n + 2n
(D) 4n 2.3n 2n

C
25.

The number of different words of three letters which can be formed from the word PROPOSAL, if a
vowel is always in the middle are
(A) 53
(B) 52
(C) 63
(D) 32

MathsExplorer.com Mathsclub.org where Mathematics is more colourful

P a g e | 62

Permutation and Combinations: Art of Counting

26.

27.

28.

29.

30.

31.

32.

The number of ways , in which 10 candidate A1, A2..A10 can be ranked so that A1 is always above A2 is
(A) 10!

(B) 10!/2

(C) 2.10!

(D) 9!

The number of numbers, that can be formed by using digits 1,2,3,4,3,2,1 , so that odd digits always
occupy odd places, is
(A) 3!4!

(B) 34

(C) 18

(D) 12

The number of positive integral solution of abc = 30 is


(A) 25

(B) 26

(C) 27

(D) 28

Total number of divisors of n = 35 . 57. 79 that are of the form 4 + 1, 0, is equal to


(A) 240

(B) 30

(C) 120

(D) 15.

Number of even proper divisors of 1260 are


(A) 24

(B) 23

(C) 36

(D) 4.

The number of proper divisors of 2p . 6q . 15r is


(A) (p + q + 1)(q + r + 1)(r + 1)

(B) (p + q + 1)(q + r + 1)(r + 1) 2

(C) (p + q) (q + r) r 2

(D) none of these

The number of odd proper divisors of 3p . 6m. .21n is


(A) (p + 1)(m + 1)(n + 1) 2

(B) (p + m + n + 1)(n + 1) 1

(C) (p + 1) (m + 1) (n + 1) 1

(D) none of these.

MathsExplorer.com Mathsclub.org where Mathematics is more colourful

P a g e | 63

Permutation and Combinations: Art of Counting


33.

34.

35.

36.

37.

The number of even proper divisors of 1008 is


(A) 23

(B) 21

(C) 20

(D) none of these.

The total number of ways in which a begger can be given at least one rupee from four 25-paise coins,
three 50 paise coins and 2 one-rupee coins is
(A) 54

(B) 50

(C) 52

(D) none of these.

Number of triangles which can be formed from 12 points out of which 7 are collinear
(A) 105

(B) 210

(C) 175

(D) 185

In a room there are 12 bulbs of the same voltage, each having a separate switch. The number of ways to
light the room with different amount of illumination.
(A) 122 1

(B) 212

(C) 212 1

(D) none of these

In an examination a candidate is required to pass four different subjects. The number of ways be can fail
is
(A) 4

(B) 10

(B) 15

(D) 24

38.

The number of six digit numbers, all digits of which are odd is
(A) 54
(B) 56
(C) 55
(D) none of thee

B
39.

The number of ways in which the digits of the number 125453752 can be rearranged such that no two
5s come together is

9!
3! 2!
(A)

7!
3! 2!
(B)
MathsExplorer.com Mathsclub.org where Mathematics is more colourful

P a g e | 64

Permutation and Combinations: Art of Counting


7
3 C. 6!

2!

(C)

(D) None

40.

The number of ways in which 5 beads, chosen from 8 different beads be threaded on to the ring is
(A) 672
(B) 1344
(C) 336
(D) none

A
41.

The number of ways in which Rs 20 can be distributed among five persons 50 that each person receives
at least Rs 3 is
(A) 63

(B) 126

(C) 252

(D) 360

B
42.

The number of words that can be formed from the letter a, b, c , d , e, f taken 3 at a times , each word
containing at least one vowel is
(A) 96

(B) 84

(C) 106

(D) None of these

A
43.

There are three copies each of 4 different books, the number of ways in which they can be arranged in a
self is

12!
(3! ) 4
(A)

11!
(3! ) 2
(B)

9!
(3! ) 2
(C)

(D) None of these

44.

The number of lines drawn through 6 points lying on a circle is


(A) 12

(B) 15

(C) 24

(D) 30

MathsExplorer.com Mathsclub.org where Mathematics is more colourful

P a g e | 65

Permutation and Combinations: Art of Counting


45.

12 persons are to be arranged to a round table if two particular persons among them are not to be side
by side, the total number of arrangement is
(A) 9(10)!

(B) 2(10)!

(C) 45(8)!

(D) 10!

46.

Four boys picked 30 apples , the number of ways in which they can divide then if all the apples are
identical is
(A) 5630

(B) 4260

(C) 5456

(D) None of these

C
47.

n and m are selected from {1, 2, 3, 4, , 20} with replacement. Number of ways selecting (m, n) such
that 7m + 7n is divisible by 10, is
(A) 25

(B) 100

(C) 625

(D) 150

48.

Number of ways in which a 10 digit number can be formed using {1, 2, 3, 4, 5, - 6}, such that sum of 1 st
and 2nd digit, 3rd and 4th digit, , so on is 10.
(A) 25

(B) 35

(C) 45

(D) 55

B
49.

A double decker bus can accommodate (u + l) passengers, u in the upper deck and l in the lower deck.
The number of ways in which the (u + l) passengers can be distributed in the two decks if r(l) perticular
passengers refuse to go in the upper deck and s(u) refuse to sit in the lower deck is
(u l r s)!
(l r )! (u s)!

(A) u-sCl - r

(C) u - sPl r

(B)

(u l)!
r! s!
(D)

MathsExplorer.com Mathsclub.org where Mathematics is more colourful

P a g e | 66

Permutation and Combinations: Art of Counting


50.

The number of integral solutions of x + y + z = 0 with x, y, z -4 is


(A) 15C7
(B) 14C12
(C) 17C15
(D) 17C10

B
51.

The number of 4 digit numbers having non-zero digits such that sum of the digits is 10 is (repetition of
digits not allowed)
(A) 42

(B) 34

(C) 84

(D) 24

52.

The number of ways of switching the network such that the


bulb glows is
(A) 61

(B) 6

(C) 7

(D) None of

these

A
53.

The number of even proper divisors of 1008 is


(A) 23

(B) 21

(C) 20

(D) 24

D
54.

The number of non-negative integral solutions of x1 + x2 + x3 + 4x4 = 20 is


(A) 520

(B) 536

(C) 720

(D) none of these

B
55.

The number of permutations of 4 letter word taken from the word EXAMINATION having two alike and
two different letters is
(A) 752
(B) 756
(C) 758

(D) 780

B
56.

Number of ways in which two smaller square can be selected from a chessboard such that they have
exactly one corner common

MathsExplorer.com Mathsclub.org where Mathematics is more colourful

P a g e | 67

Permutation and Combinations: Art of Counting


(A) 98

(B) 216

(C) 112

(D) 90

D
57.

Number of ways of distributing 10 identical objects among 8 persons(one or many persons may not be
getting any number of objects) is
(A) 810

(B) 108

17
7 C

(C)

10
8 C

(D)

C
58.

In how many ways the number 10800 be resolved as a product of two factors (A) 35
(B) 36
(C) 29
(D) 30

D
59.

The number of ways in which three distinct numbering A.P can be selected from the set
{1, 2, 3 .12} is equal to
(A) 18

(B) 24

(C) 30

(D) 15.

60.

All the students of a class send New year greetings to one another. If the postman delivers 1190
greeting cards to the students of the class. Then the number of students in it is
(A) 15
(B) 25
(C) 35
(D) 45

61.

The number of different nine digit number that can be formed from the number 223355888 by
rearranging its digits so that the odd digits occupy even positions are
(A) 16
(B) 36
(C) 60
180

62.

The number of all the odd divisors of 3600 is


(A) 45
(C) 18

63.

(B) 4
(D) 9

If a, b, c, d, e are prime integers, then the number of divisors of ab2c2de excluding 1 as a factor, is

MathsExplorer.com Mathsclub.org where Mathematics is more colourful

P a g e | 68

Permutation and Combinations: Art of Counting


(A) 94
(C) 36

64.

65.

66.

(B) 72
(D) 71

In a one day cricket match between India and West Indies, the Indians require 10 runs in the last 3 balls
to win. If any one of the scores 0, 1, 2, 3, 4, 5, 6 can be made from a ball and no wides or no balls are
bowled, then the number of different ways that batsmen make exactly 10 runs is
(A) 15

(B) 21

(C) 17

(D) none of these

The number of words of 6 letters that can be formed with the letters of the word ABRAKADABRA, if the
word begins and end with A are
(A 270

(B) 290

(C) 370

(D) 345

A polygon has 44 diagonals, then n is equal to


(A) 10
(B) 11
(C) 12
(D) 13

B
67.

Cr + 2.nCr + 1 + nCr+2 is equal to (2 r n)


(A) 2.nCr + 2
(C) n + 2Cr + 2
n

(B) n +1Cr +1
(D) none of these

C
68.

The number of five digits telephone numbers having atleast one of their digits repeated is
(A) 90,000
(B) 100000
(C) 30240
(D) 69760

D
69.

How many words can be made from the letters of the word INSURANCE, if all vowels come together
(A) 18270

(B) 17280

(C) 12780

(D) None of these

70.

If a, b, c, d, e are prime integers, then the number of divisors of ab2c2de excluding 1 as a factor, is
(A) 94

(B) 72

(C) 36

(D) 71

MathsExplorer.com Mathsclub.org where Mathematics is more colourful

P a g e | 69

Permutation and Combinations: Art of Counting


D

71.

Number of all four digit numbers having different digits formed of the digits 1, 2, 3, 4 and 5 and divisible
by 4 is
(A) 24

(B) 30

(C) 125

(D) 100

72.

Let A be the set of 4-digit numbers a1a2a3a4 where a1> a2> a3> a4, then n(A) is equal to
(A) 126

(B) 84

(C) 210

(D) none of these

73.

Set A has 3 elements and set B has 4 elements. The number of injections that can be defined from A to B
is
(A) 144

(B) 12

(C) 24

(D) 64

74.

If A and B are disjoint finite sets then number of elements in A B is


(A) n(A B)

(B) n(A B)

(C) n(A) n (B)

(D) n(A) + n (B)

75.

If x = {1, 2, 3, 4, 5, 6, 7}, then the number of proper subsets of x, containing 7, is


(A) 60

(B) 61

(C) 62

(D) 63

MathsExplorer.com Mathsclub.org where Mathematics is more colourful

P a g e | 70

Permutation and Combinations: Art of Counting


ANSWERS (Objective)
LEVEL I
1.

2.

3.

4.

5.

6.

7.

8.

9.

10.

11.

12.

13.

14.

A, B, C

15.

16.

17.

18.

19.

20.

LEVEL -II

1.

2.

3.

4.

5.

6.

7.

8.

9.

10.

11.

12.

13.

14.

15.

16.

17.

18.

19.

20.

MathsExplorer.com Mathsclub.org where Mathematics is more colourful

You might also like